How to find this series coefficients?












0












$begingroup$


I want to find the following series coefficients:
begin{equation}sum_2^infty B_{n}[sin(nx)-ncos(nx)]=sin(x)+frac{x^2}{pi}-xend{equation}



Interval is $[0,pi]$.



I tried:
begin{equation}B_{n}=frac{2}{pi}int_{0}^{pi} left(sin(nx)-ncos(nx)right)left(sin(x)+frac{x^2}{pi}-xright)dxend{equation}



So I found:



begin{equation}B_{n}=frac{2}{pi^2}left[frac{2+n^2(-2+pi)+(n^2(2+pi)-2)(-1)^n}{n^5-n^3}right]end{equation}



The problem is the denominator of $B_{n}$ which must be $n^7-n^3$ instead of
$n^5-n^3$.



The solution in my tutor's PDF is:
begin{equation}B_{n}=frac{2}{pi^2}left[frac{2+n^2(-2+pi)+(n^2(2+pi)-2)(-1)^n}{n^7-n^3}right]end{equation}



Where is the problem?










share|cite|improve this question











$endgroup$












  • $begingroup$
    $B_n$ is defined by $B_n=frac{1}{2pi}int_{-pi}^pi (sin(x)+frac{x^2}{pi}-x)sin nxrm dx = frac{1}{2npi}int_{-pi}^pi (sin(x)+frac{x^2}{pi}-x)cos nxrm dx$.
    $endgroup$
    – Nicolas FRANCOIS
    Jan 24 at 19:45










  • $begingroup$
    That is, if the interval on which the equality holds is $[-pi,pi]$. I didn't verified. But your way of computing $B_n$ is strange.
    $endgroup$
    – Nicolas FRANCOIS
    Jan 24 at 19:46










  • $begingroup$
    @NicolasFRANCOIS The interval is $[0,pi]$
    $endgroup$
    – MyBBExpert
    Jan 24 at 19:57


















0












$begingroup$


I want to find the following series coefficients:
begin{equation}sum_2^infty B_{n}[sin(nx)-ncos(nx)]=sin(x)+frac{x^2}{pi}-xend{equation}



Interval is $[0,pi]$.



I tried:
begin{equation}B_{n}=frac{2}{pi}int_{0}^{pi} left(sin(nx)-ncos(nx)right)left(sin(x)+frac{x^2}{pi}-xright)dxend{equation}



So I found:



begin{equation}B_{n}=frac{2}{pi^2}left[frac{2+n^2(-2+pi)+(n^2(2+pi)-2)(-1)^n}{n^5-n^3}right]end{equation}



The problem is the denominator of $B_{n}$ which must be $n^7-n^3$ instead of
$n^5-n^3$.



The solution in my tutor's PDF is:
begin{equation}B_{n}=frac{2}{pi^2}left[frac{2+n^2(-2+pi)+(n^2(2+pi)-2)(-1)^n}{n^7-n^3}right]end{equation}



Where is the problem?










share|cite|improve this question











$endgroup$












  • $begingroup$
    $B_n$ is defined by $B_n=frac{1}{2pi}int_{-pi}^pi (sin(x)+frac{x^2}{pi}-x)sin nxrm dx = frac{1}{2npi}int_{-pi}^pi (sin(x)+frac{x^2}{pi}-x)cos nxrm dx$.
    $endgroup$
    – Nicolas FRANCOIS
    Jan 24 at 19:45










  • $begingroup$
    That is, if the interval on which the equality holds is $[-pi,pi]$. I didn't verified. But your way of computing $B_n$ is strange.
    $endgroup$
    – Nicolas FRANCOIS
    Jan 24 at 19:46










  • $begingroup$
    @NicolasFRANCOIS The interval is $[0,pi]$
    $endgroup$
    – MyBBExpert
    Jan 24 at 19:57
















0












0








0





$begingroup$


I want to find the following series coefficients:
begin{equation}sum_2^infty B_{n}[sin(nx)-ncos(nx)]=sin(x)+frac{x^2}{pi}-xend{equation}



Interval is $[0,pi]$.



I tried:
begin{equation}B_{n}=frac{2}{pi}int_{0}^{pi} left(sin(nx)-ncos(nx)right)left(sin(x)+frac{x^2}{pi}-xright)dxend{equation}



So I found:



begin{equation}B_{n}=frac{2}{pi^2}left[frac{2+n^2(-2+pi)+(n^2(2+pi)-2)(-1)^n}{n^5-n^3}right]end{equation}



The problem is the denominator of $B_{n}$ which must be $n^7-n^3$ instead of
$n^5-n^3$.



The solution in my tutor's PDF is:
begin{equation}B_{n}=frac{2}{pi^2}left[frac{2+n^2(-2+pi)+(n^2(2+pi)-2)(-1)^n}{n^7-n^3}right]end{equation}



Where is the problem?










share|cite|improve this question











$endgroup$




I want to find the following series coefficients:
begin{equation}sum_2^infty B_{n}[sin(nx)-ncos(nx)]=sin(x)+frac{x^2}{pi}-xend{equation}



Interval is $[0,pi]$.



I tried:
begin{equation}B_{n}=frac{2}{pi}int_{0}^{pi} left(sin(nx)-ncos(nx)right)left(sin(x)+frac{x^2}{pi}-xright)dxend{equation}



So I found:



begin{equation}B_{n}=frac{2}{pi^2}left[frac{2+n^2(-2+pi)+(n^2(2+pi)-2)(-1)^n}{n^5-n^3}right]end{equation}



The problem is the denominator of $B_{n}$ which must be $n^7-n^3$ instead of
$n^5-n^3$.



The solution in my tutor's PDF is:
begin{equation}B_{n}=frac{2}{pi^2}left[frac{2+n^2(-2+pi)+(n^2(2+pi)-2)(-1)^n}{n^7-n^3}right]end{equation}



Where is the problem?







sequences-and-series






share|cite|improve this question















share|cite|improve this question













share|cite|improve this question




share|cite|improve this question








edited Jan 24 at 19:58







MyBBExpert

















asked Jan 24 at 19:39









MyBBExpertMyBBExpert

32




32












  • $begingroup$
    $B_n$ is defined by $B_n=frac{1}{2pi}int_{-pi}^pi (sin(x)+frac{x^2}{pi}-x)sin nxrm dx = frac{1}{2npi}int_{-pi}^pi (sin(x)+frac{x^2}{pi}-x)cos nxrm dx$.
    $endgroup$
    – Nicolas FRANCOIS
    Jan 24 at 19:45










  • $begingroup$
    That is, if the interval on which the equality holds is $[-pi,pi]$. I didn't verified. But your way of computing $B_n$ is strange.
    $endgroup$
    – Nicolas FRANCOIS
    Jan 24 at 19:46










  • $begingroup$
    @NicolasFRANCOIS The interval is $[0,pi]$
    $endgroup$
    – MyBBExpert
    Jan 24 at 19:57




















  • $begingroup$
    $B_n$ is defined by $B_n=frac{1}{2pi}int_{-pi}^pi (sin(x)+frac{x^2}{pi}-x)sin nxrm dx = frac{1}{2npi}int_{-pi}^pi (sin(x)+frac{x^2}{pi}-x)cos nxrm dx$.
    $endgroup$
    – Nicolas FRANCOIS
    Jan 24 at 19:45










  • $begingroup$
    That is, if the interval on which the equality holds is $[-pi,pi]$. I didn't verified. But your way of computing $B_n$ is strange.
    $endgroup$
    – Nicolas FRANCOIS
    Jan 24 at 19:46










  • $begingroup$
    @NicolasFRANCOIS The interval is $[0,pi]$
    $endgroup$
    – MyBBExpert
    Jan 24 at 19:57


















$begingroup$
$B_n$ is defined by $B_n=frac{1}{2pi}int_{-pi}^pi (sin(x)+frac{x^2}{pi}-x)sin nxrm dx = frac{1}{2npi}int_{-pi}^pi (sin(x)+frac{x^2}{pi}-x)cos nxrm dx$.
$endgroup$
– Nicolas FRANCOIS
Jan 24 at 19:45




$begingroup$
$B_n$ is defined by $B_n=frac{1}{2pi}int_{-pi}^pi (sin(x)+frac{x^2}{pi}-x)sin nxrm dx = frac{1}{2npi}int_{-pi}^pi (sin(x)+frac{x^2}{pi}-x)cos nxrm dx$.
$endgroup$
– Nicolas FRANCOIS
Jan 24 at 19:45












$begingroup$
That is, if the interval on which the equality holds is $[-pi,pi]$. I didn't verified. But your way of computing $B_n$ is strange.
$endgroup$
– Nicolas FRANCOIS
Jan 24 at 19:46




$begingroup$
That is, if the interval on which the equality holds is $[-pi,pi]$. I didn't verified. But your way of computing $B_n$ is strange.
$endgroup$
– Nicolas FRANCOIS
Jan 24 at 19:46












$begingroup$
@NicolasFRANCOIS The interval is $[0,pi]$
$endgroup$
– MyBBExpert
Jan 24 at 19:57






$begingroup$
@NicolasFRANCOIS The interval is $[0,pi]$
$endgroup$
– MyBBExpert
Jan 24 at 19:57












1 Answer
1






active

oldest

votes


















1












$begingroup$

Note that the basis functions $u_n = sin(nx) - n cos(nx)$ are orthogonal on $[0,pi]$, with $int_0^pi u_n^2(x); dx = pi (n^2+1)/2$. Thus the coefficients for a function $f(x)$ that has such a series should be
$$ b_n = frac{2}{pi(n^2+1)} int_0^pi u_n(x) f(x); dx $$
In this case, if $n ne 1$,
$$ eqalign{B_n &= frac{2}{n^2+1} int_0^pi (sin(nx)-ncos(nx))left(sin(x)+frac{x^2}{pi}-xright); dx cr
&= {frac {2,pi, left( -1 right) ^{n}{n}^{2}+2,pi,{n}^{2}+4,{n}^
{2} left( -1 right) ^{n}-4,{n}^{2}-4, left( -1 right) ^{n}+4}{{n
}^{3}pi^2, left( {n}^{4}-1 right) }}
} $$

while
$$B_1 = frac{pi^2-8}{2pi^2}$$






share|cite|improve this answer









$endgroup$













  • $begingroup$
    Thank you! What about $B_{0}$ is that $frac{1}{pi}int_{0}^{pi} (sin(x)+frac{x^2}{pi}-x)dx$ or $frac{2}{pi}int_{0}^{pi} (sin(x)+frac{x^2}{pi}-x)dx$?
    $endgroup$
    – MyBBExpert
    Jan 24 at 21:35












  • $begingroup$
    Is there a $B_0$? For $n=0$, $sin(nx) - n cos(nx) = 0$.
    $endgroup$
    – Robert Israel
    Jan 25 at 1:52










  • $begingroup$
    Hm: these $f_n$ are eigenfunctions for $f''= lambda f$ with boundary conditions $f(0) = -f'(0)$, $f(pi) = -f'(pi)$. Besides $f_n$ for positive integers $n$, the other eigenfunction is $exp(-x)$. So that's what will give you another term in the expansion..
    $endgroup$
    – Robert Israel
    Jan 25 at 2:22













Your Answer





StackExchange.ifUsing("editor", function () {
return StackExchange.using("mathjaxEditing", function () {
StackExchange.MarkdownEditor.creationCallbacks.add(function (editor, postfix) {
StackExchange.mathjaxEditing.prepareWmdForMathJax(editor, postfix, [["$", "$"], ["\\(","\\)"]]);
});
});
}, "mathjax-editing");

StackExchange.ready(function() {
var channelOptions = {
tags: "".split(" "),
id: "69"
};
initTagRenderer("".split(" "), "".split(" "), channelOptions);

StackExchange.using("externalEditor", function() {
// Have to fire editor after snippets, if snippets enabled
if (StackExchange.settings.snippets.snippetsEnabled) {
StackExchange.using("snippets", function() {
createEditor();
});
}
else {
createEditor();
}
});

function createEditor() {
StackExchange.prepareEditor({
heartbeatType: 'answer',
autoActivateHeartbeat: false,
convertImagesToLinks: true,
noModals: true,
showLowRepImageUploadWarning: true,
reputationToPostImages: 10,
bindNavPrevention: true,
postfix: "",
imageUploader: {
brandingHtml: "Powered by u003ca class="icon-imgur-white" href="https://imgur.com/"u003eu003c/au003e",
contentPolicyHtml: "User contributions licensed under u003ca href="https://creativecommons.org/licenses/by-sa/3.0/"u003ecc by-sa 3.0 with attribution requiredu003c/au003e u003ca href="https://stackoverflow.com/legal/content-policy"u003e(content policy)u003c/au003e",
allowUrls: true
},
noCode: true, onDemand: true,
discardSelector: ".discard-answer"
,immediatelyShowMarkdownHelp:true
});


}
});














draft saved

draft discarded


















StackExchange.ready(
function () {
StackExchange.openid.initPostLogin('.new-post-login', 'https%3a%2f%2fmath.stackexchange.com%2fquestions%2f3086270%2fhow-to-find-this-series-coefficients%23new-answer', 'question_page');
}
);

Post as a guest















Required, but never shown

























1 Answer
1






active

oldest

votes








1 Answer
1






active

oldest

votes









active

oldest

votes






active

oldest

votes









1












$begingroup$

Note that the basis functions $u_n = sin(nx) - n cos(nx)$ are orthogonal on $[0,pi]$, with $int_0^pi u_n^2(x); dx = pi (n^2+1)/2$. Thus the coefficients for a function $f(x)$ that has such a series should be
$$ b_n = frac{2}{pi(n^2+1)} int_0^pi u_n(x) f(x); dx $$
In this case, if $n ne 1$,
$$ eqalign{B_n &= frac{2}{n^2+1} int_0^pi (sin(nx)-ncos(nx))left(sin(x)+frac{x^2}{pi}-xright); dx cr
&= {frac {2,pi, left( -1 right) ^{n}{n}^{2}+2,pi,{n}^{2}+4,{n}^
{2} left( -1 right) ^{n}-4,{n}^{2}-4, left( -1 right) ^{n}+4}{{n
}^{3}pi^2, left( {n}^{4}-1 right) }}
} $$

while
$$B_1 = frac{pi^2-8}{2pi^2}$$






share|cite|improve this answer









$endgroup$













  • $begingroup$
    Thank you! What about $B_{0}$ is that $frac{1}{pi}int_{0}^{pi} (sin(x)+frac{x^2}{pi}-x)dx$ or $frac{2}{pi}int_{0}^{pi} (sin(x)+frac{x^2}{pi}-x)dx$?
    $endgroup$
    – MyBBExpert
    Jan 24 at 21:35












  • $begingroup$
    Is there a $B_0$? For $n=0$, $sin(nx) - n cos(nx) = 0$.
    $endgroup$
    – Robert Israel
    Jan 25 at 1:52










  • $begingroup$
    Hm: these $f_n$ are eigenfunctions for $f''= lambda f$ with boundary conditions $f(0) = -f'(0)$, $f(pi) = -f'(pi)$. Besides $f_n$ for positive integers $n$, the other eigenfunction is $exp(-x)$. So that's what will give you another term in the expansion..
    $endgroup$
    – Robert Israel
    Jan 25 at 2:22


















1












$begingroup$

Note that the basis functions $u_n = sin(nx) - n cos(nx)$ are orthogonal on $[0,pi]$, with $int_0^pi u_n^2(x); dx = pi (n^2+1)/2$. Thus the coefficients for a function $f(x)$ that has such a series should be
$$ b_n = frac{2}{pi(n^2+1)} int_0^pi u_n(x) f(x); dx $$
In this case, if $n ne 1$,
$$ eqalign{B_n &= frac{2}{n^2+1} int_0^pi (sin(nx)-ncos(nx))left(sin(x)+frac{x^2}{pi}-xright); dx cr
&= {frac {2,pi, left( -1 right) ^{n}{n}^{2}+2,pi,{n}^{2}+4,{n}^
{2} left( -1 right) ^{n}-4,{n}^{2}-4, left( -1 right) ^{n}+4}{{n
}^{3}pi^2, left( {n}^{4}-1 right) }}
} $$

while
$$B_1 = frac{pi^2-8}{2pi^2}$$






share|cite|improve this answer









$endgroup$













  • $begingroup$
    Thank you! What about $B_{0}$ is that $frac{1}{pi}int_{0}^{pi} (sin(x)+frac{x^2}{pi}-x)dx$ or $frac{2}{pi}int_{0}^{pi} (sin(x)+frac{x^2}{pi}-x)dx$?
    $endgroup$
    – MyBBExpert
    Jan 24 at 21:35












  • $begingroup$
    Is there a $B_0$? For $n=0$, $sin(nx) - n cos(nx) = 0$.
    $endgroup$
    – Robert Israel
    Jan 25 at 1:52










  • $begingroup$
    Hm: these $f_n$ are eigenfunctions for $f''= lambda f$ with boundary conditions $f(0) = -f'(0)$, $f(pi) = -f'(pi)$. Besides $f_n$ for positive integers $n$, the other eigenfunction is $exp(-x)$. So that's what will give you another term in the expansion..
    $endgroup$
    – Robert Israel
    Jan 25 at 2:22
















1












1








1





$begingroup$

Note that the basis functions $u_n = sin(nx) - n cos(nx)$ are orthogonal on $[0,pi]$, with $int_0^pi u_n^2(x); dx = pi (n^2+1)/2$. Thus the coefficients for a function $f(x)$ that has such a series should be
$$ b_n = frac{2}{pi(n^2+1)} int_0^pi u_n(x) f(x); dx $$
In this case, if $n ne 1$,
$$ eqalign{B_n &= frac{2}{n^2+1} int_0^pi (sin(nx)-ncos(nx))left(sin(x)+frac{x^2}{pi}-xright); dx cr
&= {frac {2,pi, left( -1 right) ^{n}{n}^{2}+2,pi,{n}^{2}+4,{n}^
{2} left( -1 right) ^{n}-4,{n}^{2}-4, left( -1 right) ^{n}+4}{{n
}^{3}pi^2, left( {n}^{4}-1 right) }}
} $$

while
$$B_1 = frac{pi^2-8}{2pi^2}$$






share|cite|improve this answer









$endgroup$



Note that the basis functions $u_n = sin(nx) - n cos(nx)$ are orthogonal on $[0,pi]$, with $int_0^pi u_n^2(x); dx = pi (n^2+1)/2$. Thus the coefficients for a function $f(x)$ that has such a series should be
$$ b_n = frac{2}{pi(n^2+1)} int_0^pi u_n(x) f(x); dx $$
In this case, if $n ne 1$,
$$ eqalign{B_n &= frac{2}{n^2+1} int_0^pi (sin(nx)-ncos(nx))left(sin(x)+frac{x^2}{pi}-xright); dx cr
&= {frac {2,pi, left( -1 right) ^{n}{n}^{2}+2,pi,{n}^{2}+4,{n}^
{2} left( -1 right) ^{n}-4,{n}^{2}-4, left( -1 right) ^{n}+4}{{n
}^{3}pi^2, left( {n}^{4}-1 right) }}
} $$

while
$$B_1 = frac{pi^2-8}{2pi^2}$$







share|cite|improve this answer












share|cite|improve this answer



share|cite|improve this answer










answered Jan 24 at 20:24









Robert IsraelRobert Israel

327k23216469




327k23216469












  • $begingroup$
    Thank you! What about $B_{0}$ is that $frac{1}{pi}int_{0}^{pi} (sin(x)+frac{x^2}{pi}-x)dx$ or $frac{2}{pi}int_{0}^{pi} (sin(x)+frac{x^2}{pi}-x)dx$?
    $endgroup$
    – MyBBExpert
    Jan 24 at 21:35












  • $begingroup$
    Is there a $B_0$? For $n=0$, $sin(nx) - n cos(nx) = 0$.
    $endgroup$
    – Robert Israel
    Jan 25 at 1:52










  • $begingroup$
    Hm: these $f_n$ are eigenfunctions for $f''= lambda f$ with boundary conditions $f(0) = -f'(0)$, $f(pi) = -f'(pi)$. Besides $f_n$ for positive integers $n$, the other eigenfunction is $exp(-x)$. So that's what will give you another term in the expansion..
    $endgroup$
    – Robert Israel
    Jan 25 at 2:22




















  • $begingroup$
    Thank you! What about $B_{0}$ is that $frac{1}{pi}int_{0}^{pi} (sin(x)+frac{x^2}{pi}-x)dx$ or $frac{2}{pi}int_{0}^{pi} (sin(x)+frac{x^2}{pi}-x)dx$?
    $endgroup$
    – MyBBExpert
    Jan 24 at 21:35












  • $begingroup$
    Is there a $B_0$? For $n=0$, $sin(nx) - n cos(nx) = 0$.
    $endgroup$
    – Robert Israel
    Jan 25 at 1:52










  • $begingroup$
    Hm: these $f_n$ are eigenfunctions for $f''= lambda f$ with boundary conditions $f(0) = -f'(0)$, $f(pi) = -f'(pi)$. Besides $f_n$ for positive integers $n$, the other eigenfunction is $exp(-x)$. So that's what will give you another term in the expansion..
    $endgroup$
    – Robert Israel
    Jan 25 at 2:22


















$begingroup$
Thank you! What about $B_{0}$ is that $frac{1}{pi}int_{0}^{pi} (sin(x)+frac{x^2}{pi}-x)dx$ or $frac{2}{pi}int_{0}^{pi} (sin(x)+frac{x^2}{pi}-x)dx$?
$endgroup$
– MyBBExpert
Jan 24 at 21:35






$begingroup$
Thank you! What about $B_{0}$ is that $frac{1}{pi}int_{0}^{pi} (sin(x)+frac{x^2}{pi}-x)dx$ or $frac{2}{pi}int_{0}^{pi} (sin(x)+frac{x^2}{pi}-x)dx$?
$endgroup$
– MyBBExpert
Jan 24 at 21:35














$begingroup$
Is there a $B_0$? For $n=0$, $sin(nx) - n cos(nx) = 0$.
$endgroup$
– Robert Israel
Jan 25 at 1:52




$begingroup$
Is there a $B_0$? For $n=0$, $sin(nx) - n cos(nx) = 0$.
$endgroup$
– Robert Israel
Jan 25 at 1:52












$begingroup$
Hm: these $f_n$ are eigenfunctions for $f''= lambda f$ with boundary conditions $f(0) = -f'(0)$, $f(pi) = -f'(pi)$. Besides $f_n$ for positive integers $n$, the other eigenfunction is $exp(-x)$. So that's what will give you another term in the expansion..
$endgroup$
– Robert Israel
Jan 25 at 2:22






$begingroup$
Hm: these $f_n$ are eigenfunctions for $f''= lambda f$ with boundary conditions $f(0) = -f'(0)$, $f(pi) = -f'(pi)$. Besides $f_n$ for positive integers $n$, the other eigenfunction is $exp(-x)$. So that's what will give you another term in the expansion..
$endgroup$
– Robert Israel
Jan 25 at 2:22




















draft saved

draft discarded




















































Thanks for contributing an answer to Mathematics Stack Exchange!


  • Please be sure to answer the question. Provide details and share your research!

But avoid



  • Asking for help, clarification, or responding to other answers.

  • Making statements based on opinion; back them up with references or personal experience.


Use MathJax to format equations. MathJax reference.


To learn more, see our tips on writing great answers.




draft saved


draft discarded














StackExchange.ready(
function () {
StackExchange.openid.initPostLogin('.new-post-login', 'https%3a%2f%2fmath.stackexchange.com%2fquestions%2f3086270%2fhow-to-find-this-series-coefficients%23new-answer', 'question_page');
}
);

Post as a guest















Required, but never shown





















































Required, but never shown














Required, but never shown












Required, but never shown







Required, but never shown

































Required, but never shown














Required, but never shown












Required, but never shown







Required, but never shown







Popular posts from this blog

Can a sorcerer learn a 5th-level spell early by creating spell slots using the Font of Magic feature?

Does disintegrating a polymorphed enemy still kill it after the 2018 errata?

A Topological Invariant for $pi_3(U(n))$